recategorized by
652 views

1 Answer

3 votes
3 votes
$\underline{\textbf {Answer: C}}$

$\underline{\mathbf{Solution:}}$

On differentiating $\mathbf{f(x)}$ $\mathbf{w.r.t.}\;$ $\mathbf x$, we get:

$$\mathrm {f'(x) = 0 + 1 +\underbrace{ x^1 + x^2 + x^3+\ldots+x^{2017}}_{\text{ = 2017 terms}}}$$

$$\mathrm{\therefore f'(x) = 1 + x^1 + x^2 + x^3 +\ldots+ x^{2017}}$$

Now substitute $\color{green} {\mathrm{x = 1}}$, we get:

$$\mathrm{f'(x) = 1 + \underbrace{1 + 1 + \ldots+ 1}_\text{ = 2017 times 1 = $2017\times1 = 2017$}}$$

$$\mathrm{\Rightarrow f'(x) = 1 + 2017 = 2018}$$

$\therefore \textbf {C}$ is the right option.
edited by

Related questions

1 votes
1 votes
1 answer
1
gatecse asked Sep 18, 2019
381 views
Let $f’(x)=4x^3-3x^2+2x+k,$ $f(0)=1$ and $f(1)=4.$ Then $f(x)$ is equal to$4x^4-3x^3+2x^2+x+1$$x^4-x^3+x^2+2x+1$$x^4-x^3+x^2+2(x+1)$none of these
0 votes
0 votes
1 answer
2
gatecse asked Sep 18, 2019
405 views
Let $[x]$ denote the largest integer less than or equal to $x.$ The number of points in the open interval $(1,3)$ in which the function $f(x)=a^{[x^2]},a\gt1$ is not diff...
0 votes
0 votes
0 answers
3
gatecse asked Sep 18, 2019
349 views
Let $f(x)=(x-1)(x-2)(x-3)g(x); \: x\in \mathbb{R}$ where $g$ is twice differentiable function. Thenthere exists $y\in(1,3)$ such that $f’’(y)=0.$there exists $y\in(1,...
0 votes
0 votes
1 answer
4
gatecse asked Sep 18, 2019
304 views
Let $f(x)=e^{-\big( \frac{1}{x^2-3x+2} \big) };x\in \mathbb{R} \: \: \& x \notin \{1,2\}$. Let $a=\underset{n \to 1^+}{\lim}f(x)$ and $b=\underset{x \to 1^-}{\lim} f(x)$....